¿Por qué una región infinita de campo magnético en movimiento ejercería una fuerza eléctrica?

Estoy confundido acerca de la fuerza de Lorentz en relación con el marco de referencia. Hay muchas preguntas sobre eso aquí, pero todavía no lo entiendo. He tratado de dividirlo en el ejemplo más básico.

Digamos que tenemos una sola carga moviéndose hacia la pantalla con velocidad v entre dos barras magnéticas infinitamente largas. La ley de fuerza de Lorentz nos dice que la carga experimentará una fuerza lateral.

diagrama de carga en movimiento

Hasta ahora, todo bien. Pero ahora cambio el marco de referencia a uno donde los imanes se mueven a gran velocidad. v fuera de la pantalla y la partícula cargada permanece estacionaria. El movimiento relativo es el mismo, así que espero la misma fuerza.

Como la partícula cargada no se mueve, no hay fuerza de Lorentz.

Las respuestas a las otras preguntas lo explican con el campo eléctrico. ¿De dónde viene el campo eléctrico en el segundo marco de referencia?

  • Los imanes no tienen carga neta, por lo que no puede ser la Ley de Gauss, ¿verdad?

  • Los dos imanes ahora se mueven a lo largo del eje en el que tienen una longitud infinita, por lo que el movimiento realmente no cambia nada observable. Bien podrían ser estáticos. En particular, el campo magnético no cambia, por lo que la ley de inducción de Faraday tampoco debería crear un campo eléctrico.

"Los dos imanes ahora se mueven a lo largo del eje en el que tienen una longitud infinita, por lo que el movimiento realmente no cambia nada observable. Bien podrían ser estáticos". Eso no es cierto.
Estás atravesando líneas de fuerza magnéticas, que son observables: upload.wikimedia.org/wikipedia/commons/5/57/Magnet0873.png . La única razón por la que no los está observando es porque ha decidido no poner virutas de hierro en su configuración.
Trato de evitar las virutas de hierro. Son un dolor para limpiar después. Sin realizar la transformación de Lorentz, el vector del campo magnético en la posición de la carga es siempre hacia abajo y tampoco cambia de magnitud. De hecho, el campo magnético sería estático desde el marco de referencia de la carga. Ahora, al hacer la transformación como uno tiene que hacerlo, no tengo idea de cómo se ve el resultado. Pero va a haber un campo eléctrico de alguna manera. Todavía espero una pista sobre cómo podría verse eso.
@CuriousOne Las líneas de campo magnético no son observables. Son una herramienta de diagramas útil para mostrar el aspecto de un campo vectorial al trazar algunas rutas seleccionadas a través de ellos. La única razón por la que las virutas de hierro muestran líneas de campo es porque se agrupan de forma natural.

Respuestas (2)

No se puede escapar de la relatividad especial, que es la que "unifica" los fenómenos eléctricos y magnéticos. El campo electromagnético es realmente un campo único (en realidad, un tensor, pero no se preocupe por eso), y sus componentes se mezclan en la transformada de Lorentz de manera similar a como se mezclan los componentes x, y, z de un campo vectorial habitual. juntos en una rotación.

Aquí hay dos formas de encontrar el campo eléctrico: puede realizar una transformada cinemática de Lorentz para obtener las cargas/corrientes de origen en el marco móvil. También puede obtener el componente del campo eléctrico directamente de la transformada de Lorentz del campo electromagnético.

Transformación de Lorentz en fuentes

Un objeto en movimiento parece contraído en longitud para los observadores: L = L 0 / γ , dónde L 0 es la longitud del objeto su marco de reposo y γ es el factor de Lorentz asociado con la velocidad del objeto:

γ = 1 1 v 2 / C 2

En consecuencia, si el objeto está cargado, la densidad de carga aumenta cuando está en movimiento:

ρ = γ ρ 0

El campo magnético en el marco de reposo surge de la magnetización METRO (densidad de dipolo magnético) de los imanes, que apunta hacia arriba a lo largo del eje NS. Cada dipolo magnético es efectivamente un pequeño bucle de corriente con corriente j = ρ tu , dónde tu es la velocidad tangencial alrededor del eje NS.

ingrese la descripción de la imagen aquí

Luego, en el marco en movimiento, la velocidad tu no es uniforme alrededor de la espira, y las densidades de carga en los puntos de la espira que fluyen con y en contra de la dirección del movimiento son diferentes:

tu ± = tu ± v 1 ± tu v / C 2

ρ ± = γ ± ρ 0 = ρ 0 1 tu ± 2 / C 2

Por lo tanto tu + > tu , lo que significa ρ + > ρ , por lo que cada bucle de corriente (dipolo magnético) adquiere un momento dipolar eléctrico . Cada dipolo magnético apuntaba hacia arriba, por lo que cada dipolo eléctrico apunta hacia la derecha. Esta es la fuente del campo eléctrico en el marco móvil.

Transformada de Lorentz de campos E y B

Si comienza con campos mi , B , y queremos que los campos en un marco inercial se muevan a una velocidad relativa v , los nuevos campos son:

mi = mi B = B mi = γ ( mi + v × B ) B = γ ( B 1 C 2 v × mi )

En nuestro caso mi = 0 y B = 0 , por lo que lo anterior se simplifica a

mi = γ v × B B = γ B

Si el B el campo está arriba (de S a N) y v está en la página, esto da una mi campo apuntado a la derecha.

Esta respuesta no me satisface. Todavía no explicaste la fuente del campo E después de la transformación. En cambio, digamos que comenzó en el segundo marco de referencia donde la carga estaba estacionaria. En ese cuadro, la carga no se movería y al transformar los campos al primer cuadro encontrarías que la carga aún no se mueve. Esto es inconsistente.
Seguramente las ecuaciones de Maxwell deben cumplirse en AMBOS marcos de referencia. Sin embargo, ¿cómo se satisfacen en el segundo marco de referencia donde dices que hay un campo eléctrico? De acuerdo con las ecuaciones de Maxwell, un campo eléctrico solo puede existir si hay una densidad de carga o si cambia B para producirlo. No cambia B ni la densidad de carga, por lo que parece que las ecuaciones de Maxwell predicen un campo E que es cero en todas partes.
Lo descubrí: las ecuaciones de Maxwell no garantizan que E sea cero. Solo garantizan que curl E y div E sean cero. Esto no define de forma única E sin condiciones de contorno. E podría ser un campo uniforme, por ejemplo. Entonces el punto es: E puede existir sin ninguna fuente.
@hddh Lo tienes. La extensión infinita de las barras agrega cierta confusión porque no hay una fuente de carga o, de manera equivalente, está "en el infinito". Si las barras fueran finitas, tendrían una carga superficial en cada extremo del marco móvil, generando el campo eléctrico del dipolo. Sin embargo, las ecuaciones de Maxwell están bien con tener un campo E sin fuentes en el caso infinito, solo tiene una forma muy restringida.

Si piensa que un imán está formado por dipolos individuales todos alineados entre sí (esto es lo que sucede en los imanes de barra de hierro o neodimio), entonces puede preguntar "¿cómo se transforma cada dipolo bajo la transformación de Lorentz?" Bajo la transformación de Lorentz, los dipolos magnéticos se convierten en dipolos eléctricos. El campo eléctrico de los dipolos eléctricos provoca la fuerza de Lorentz en este marco.

Con el electromagnetismo, cada vez que pregunto "¿por qué?" Demasiado a menudo, me golpean la relatividad especial y la cuántica :-) Espero que tengas razón, pero 16 páginas de ecuaciones son demasiado para mí en este momento. Tal vez podría responder a su propia formulación de mi pregunta. ¿Cómo sería el campo eléctrico generado por los dipolos eléctricos?
Si los dos marcos de referencia se mueven con v entre sí, los campos eléctricos difieren en un término v x B (suponiendo v << c ). Si podemos calcular el campo magnético en un marco, el efecto sobre el campo eléctrico en el otro se calcula fácilmente. El propio campo magnético no se ve afectado significativamente.